0 Daumen
91 Aufrufe

Sei

\( A=\left(\begin{array}{lll} 3 & 0 & 1 \\ 0 & 1 & 2 \\ 0 & 3 & 4 \end{array}\right) . \)

Wir fassen \( A \) als Matrix in \( \mathbb{F}_{p} \) auf, wobei \( p \) eine beliebige Primzahl ist. Bestimmen Sie alle \( p \), für die \( A \) in \( M_{3}\left(\mathbb{F}_{p}\right) \) invertierbar ist.

Kann man bei dieser bzw. bei solchen Aufgaben einfach die Determinante bestimmen und dann det(A) mod p und falls det(A) mod p = 0 -> nicht invertierbar. Und sonst invertierbar.

Avatar von

Da hast du doch schon eine gute Idee beschrieben.

1 Antwort

0 Daumen

Dein Ansatz ist genau richtig.

Es gilt \(\det A=-6\), also ist \(\det A_2 \equiv_2 0\), \(\det A_3 \equiv_3 0\) und \(\det A_p \not\equiv_p 0\) für alle primen \(p\neq 2,3\). Wenn du jetzt begründest (oder die VL an der korrekten Stelle zitierst), wieso in \(\mathbb{F}_p\) auch Invertierbarkeit von Matrizen genau dann gegeben ist, wenn die Determinante nicht \(0\) ist, bist du fertig.

Avatar von

Ein anderes Problem?

Stell deine Frage

Ähnliche Fragen

Willkommen bei der Mathelounge! Stell deine Frage einfach und kostenlos

x
Made by a lovely community